2.II.32D

Principles of Quantum Mechanics | Part II, 2007

Let sm|s m\rangle denote the combined spin eigenstates for a system of two particles, each with spin 1. Derive expressions for all states with m=sm=s in terms of product states.

Given that the particles are identical, and that the spatial wavefunction describing their relative position has definite orbital angular momentum \ell, show that +s\ell+s must be even. Suppose that this two-particle state is known to arise from the decay of a single particle, XX, also of spin 1. Assuming that total angular momentum and parity are conserved in this process, find the values of \ell and ss that are allowed, depending on whether the intrinsic parity of XX is even or odd.

[You may set =1\hbar=1 and use J±jm=(jm)(j±m+1)jm±1.J_{\pm}|j m\rangle=\sqrt{(j \mp m)(j \pm m+1)}|j m \pm 1\rangle . ]

Typos? Please submit corrections to this page on GitHub.